Torque Question

This forum made possible through the generous support of SDN members, donors, and sponsors. Thank you.

TheRealAngeleno

Full Member
10+ Year Member
Joined
Oct 10, 2010
Messages
120
Reaction score
0
Hey guys so I came across this torque question and I can't seem to understand the answer explanation for it so I wanted to get different perspectives on how others view the answer to this question. Thanks a lot!

Which change will NOT increase the torque on a lever system?
A. Additional weight is added to the load
B. The load is moved further from the fulcrum
C. The bar is extended on the side of the load
D. The bar is extended on the side opposite of the load

The answer is D.

Members don't see this ad.
 
I'd say there's not enough information given. You'd have to know the state of the lever prior to the change. If the lever is at equilibrium and you increase the length of the bar on the side opposite the load, you're going to increase torque.
 
I'd say there's not enough information given. You'd have to know the state of the lever prior to the change. If the lever is at equilibrium and you increase the length of the bar on the side opposite the load, you're going to increase torque.

Ah I see. So assuming this system was in equilibrium prior to the change, increasing the length of the bar on the opposite side moves the fulcrum away from the load thus increasing the moment arm and the torque.
 
Which change will NOT increase the torque on a lever system?
A. Additional weight is added to the load

If the downward force is increased and Torque is rFsin(theta) or rF,
than increasing F will increase Torque.

B. The load is moved further from the fulcrum

Similarly, if you increase the radius vector, you'll increase the Torque.
By increasing the distance of the weight from the fulcrum, you're increasing the radius vector.

C. The bar is extended on the side of the load

If you extend the bar on the same side the load is on, the center of mass will shift to the side the load is on. So if for instance, the center of mass of the bar was acting at the fulcrum originally (here torque = 0 Nm). Let's say the load is on the left side of the fulcrum. If you increase the length of the bar in that direction, you're also shifting the center of mass in that direction. Therefore, the total CCW torque will increase.

D. The bar is extended on the side opposite of the load

Shifting the center of mass (of the bar) in the opposite direction would result in a torque opposing the load. As a result, the net torque would be lower.
 
Members don't see this ad :)
Ah I see. So assuming this system was in equilibrium prior to the change, increasing the length of the bar on the opposite side moves the fulcrum away from the load thus increasing the moment arm and the torque.

The fulcrum is a designated point. For this question, you're suppose to assume that it isn't changing. The fulcrum position doesn't change. However, if you increase the length of the bar, the center of mass will certainly shift. Also, assuming the bar has a mass that's uniform throughout, we can simply say that the center of mass of the bar acts at the center of the bar.

If the bar was say 4 meters - the center of mass would be at the 2 meter point.
If the bar was increased to say, 8 meters - the center of mass would be at the 4 meter point.

Does that make sense?
 
The fulcrum is a designated point. For this question, you're suppose to assume that it isn't changing. The fulcrum position doesn't change. However, if you increase the length of the bar, the center of mass will certainly shift. Also, assuming the bar has a mass that's uniform throughout, we can simply say that the center of mass of the bar acts at the center of the bar.

If the bar was say 4 meters - the center of mass would be at the 2 meter point.
If the bar was increased to say, 8 meters - the center of mass would be at the 4 meter point.

Does that make sense?

Okay yea that makes a lot more sense. Thanks a lot.
 
Ah I see. So assuming this system was in equilibrium prior to the change, increasing the length of the bar on the opposite side moves the fulcrum away from the load thus increasing the moment arm and the torque.


You don't move the fulcrum. The assumption, in this case, is that the bar has mass. Increasing the length of the bar adds more mass on one side of the fulcrum and thus adds torque (opposing that of the load in this case). The net torque would be the torques on one side of the fulcrum summed with the torques on the other side of the fulcrum (vector quantities, so the magnitudes will subtract).

They want you to make the assumption that the system isn't in equilibrium - that the torques on the side of the load are greater than the torques on the side opposite the load. This would mean that adding mass, and thus torque, opposite the load would decrease the net torque, by counteracting some of the torque of the load. I don't think that's a reasonable assumption, but I could probably realize that that's the assumption they expect you to make. I can't say for sure though since I saw the answer before I finished reading the question.
 
Top